LSAT and Law School Admissions Forum

Get expert LSAT preparation and law school admissions advice from PowerScore Test Preparation.

 Administrator
PowerScore Staff
  • PowerScore Staff
  • Posts: 8919
  • Joined: Feb 02, 2011
|
#26528
Complete Question Explanation

Weaken—CE. The correct answer choice is (B)

The Dietician argues that North Americans could become healthier without cutting fat intake simply by drinking more red wine. This conclusion is based on the observation that French people consume high quantities of fat, however the effects of this are mitigated by the high quantities of red wine that they also consume. What is important to note however is that the stimulus shifts from a discussion of heart disease to a conclusion about overall health. So while it could be true that consuming more red wine reduces incidences of heart disease, to weaken this conclusion you should find an answer that shows that consuming more red wine may not actually be “healthier.”

Answer choice (A): Despite the fact that French men have a higher rate of heart disease than French women, the overall incidence is still as low as the stimulus suggests. Thus this answer choice does not attack the conclusion.

Answer choice (B): This is the correct answer choice. It shows that drinking more red wine by North Americans would likely lead to liver problems and other illnesses, and therefore may not result in better overall health.

Answer choice (C): The stimulus is only concerned with the overall incidence and the general trends among French people. Just because there are exceptions to that trend does not attack the notion that drinking more red wine will make North Americans healthier.

Answer choice (D): The conclusion is not about the “healthiest” way to live, only about the notion that drinking more red wine can make North Americans “healthier.” The fact that there may be an even healthier way to live does not attack the relationship presented in this conclusion.

Answer choice (E): The Paradox mentioned (and the parallel between the French and North Americans) is in part due to the high fat consumption of the French, so these other regions with lower incidence of heart disease may not meet the criteria given in the stimulus.
 JayI
  • Posts: 4
  • Joined: Jul 27, 2017
|
#37908
Hi,

I am having trouble with this question. I diagrammed the causal relationship as:

French: red wine ---mitigates---> Ill effects of Fat
North Americans: Healthier without cutting fat --> Drink more red wine.

So I attempted to attack the "red wine mitigates ill effects of fat" relationship which led me to A.

Can you please let me know what I did wrong in the causal relationship? Also the stimulus has no evidence that exercise will cause positive health effects.

Thank you.
 AthenaDalton
PowerScore Staff
  • PowerScore Staff
  • Posts: 296
  • Joined: May 02, 2017
|
#37955
Hi Jay,

I think you were actually on the right track with your cause/effect reasoning, but you focused on attacking a premise (that drinking red wine decreases heart disease) instead of attacking the main conclusion (drinking more red wine will make Americans healthier).

Answer choice (A) is incorrect because it makes a comparison between French men and French women (as opposed to, say French people and American people). We know from the stimulus that French people have a low incidence of heart disease "overall" so we're less concerned with whether French men or women suffer from a greater incidence of heart disease. Perhaps men from every country are more likely to suffer from heart disease than women -- it still doesn't undermine the fact that French people overall enjoy better heart health than most of the world.

Answer choice (C) is not an obvious pick at first glance because it focuses on "health" generally as opposed to "heart health" which is the main focus of the dietician's argument. However, it does directly attack the relationship of drinking more red wine :arrow: better health.

Answer choice (C) tells us that if Americans drink more red wine, they will suffer from a greater incidence of liver disease. This would produce a negative effect on their health, and therefore would undermine the conclusion that red wine :arrow: health.

You're also right that the stimulus doesn't include anything about exercise, but this is not essential in finding the right answer. The call of the question says to take every statement "as true," whether or not a fact in an answer choice was mentioned in the stimulus. So if an answer choice says that exercise produces health benefits, just accept it as true and see how it effects the argument.

I hope this clears things up. Good luck studying! :)
 lsatnoobie
  • Posts: 52
  • Joined: Sep 18, 2017
|
#40348
Is E wrong simply because the stimulus only discussed North America and France, and not "many other regions"?

I was able to select B by showing that even when the cause occurs, the effect does not. More wine will not equal healthier body. But I can't seem to methodologically rule out E...

Thank you!
 Adam Tyson
PowerScore Staff
  • PowerScore Staff
  • Posts: 5153
  • Joined: Apr 14, 2011
|
#40355
There are two ways to knock out answer E, noobie. One is to recognize that it fails to address a key issue in the conclusion, that of "healthier". Lower heart disease is great, but are they healthier? The second is to see that it also fails to address the other key issue in the conclusion, that of "without cutting fat intake". Do those other regions eat the same amount of fat as the French or North Americans do? Missing both of those key ingredients consigns this answer to the scrap heap.

Does it matter that it brings up other regions? Not at all! A perfectly good answer could have done that as well, such as:

"Many other regions eat as much fat and drink as much red wine as the French do, and the people there are, on average, less healthy than French people are." This might weaken the argument by showing that where the cause (drinking red wine) is present, the effect (being healthier) is not present. New information about other regions is fine, if it is presented in a way that accomplishes your goal (which in this case is to weaken the claim that North Americans would be healthier if they increased their intake of red wine).

Keep up the good work! Nice job picking B, for exactly the right reason!
 Clinton_Lowe
  • Posts: 1
  • Joined: Jul 12, 2018
|
#47842
I understand why (B) is the correct Ans. choice, but I’m still having a hard time eliminating (D) completely.
 Malila Robinson
PowerScore Staff
  • PowerScore Staff
  • Posts: 296
  • Joined: Feb 01, 2018
|
#47850
Hi Clinton_Lowe,
To eliminate D you can focus on the degree of certainty in the answer. "...the healthiest way to decrease the chance of heart disease is to exercise and keep a diet low in fat."
Ok, let's just agree for a moment that that is the healthiest way to decrease the chance of heart disease. If that is true, would it hurt an argument that says: "...if you want to be healthier without cutting fat intake, drink more red wine"?That is where the problem with D pops up. D argues for something that the argument has specifically discarded. Think of it like someone saying: "Well, obviously you would be healthier if you stopped eating all that fatty food, but since I know you won't do that, you can be healthier than you are right now if you just drink some red wine with your fatty food." In that case D doesn't hurt the argument because what has been presented in D has already been dismissed by the argument.
Hope that helps!
-Malila
 gen2871
  • Posts: 47
  • Joined: Jul 01, 2018
|
#47950
Hi, I have hard time diagraming the conclusion because it has a sufficient and necessary indicators (if, and without).
My inclination would be healthier without cutting fat intake :arrow: drink more red wine.

Answer choice B starts with a greater intake of red wine actually increase the chance of liver problems, and this is a direct attach on the conclusion. Hence it is the right answer choice.

But can someone be kindly advise me how to distinguish the sufficient and necessary condition with multiple indicators scernaio since I have seen a lot of these.

Thank you! :x :x
 Malila Robinson
PowerScore Staff
  • PowerScore Staff
  • Posts: 296
  • Joined: Feb 01, 2018
|
#49827
Hi gen2871,
Can you give us a bit more info as to what you mean by clarifying "how to distinguish the sufficient and necessary condition with multiple indicators scenario"?
The way you diagrammed the conclusion was correct, and it seems that it got you to the correct answer, so I'm not quite sure what your question relates to.
-Malila

Get the most out of your LSAT Prep Plus subscription.

Analyze and track your performance with our Testing and Analytics Package.